TEXES EC-6

Pataasin ang iyong marka sa homework at exams ngayon gamit ang Quizwiz!

Which of the following is the official state song of Texas? "Texas, Our Texas" "The Yellow Rose of Texas" "Deep in the Heart of Texas" "If You're Ever Down In Texas, Look Me Up"

"Texas, Our Texas"

A group of fourth-grade students wished to test a hypothesis that adding mass to a cart rolled down a ramp would increase the distance the cart would travel along the floor. To do this, they designed and carried out the following experiment using a toy cart with a mass of 200 g, a small ramp and several 25 g masses. 1) Make a line across the ramp near the top. 2) Line up the front wheels of the cart with the line. 3) Let go of the cart without pushing it. 4) When the cart stops, measure the distance from the bottom of the ramp to the place where the front wheels stopped, write down this distance. 5) Add a 25 g mass to the cart. 6) Repeat until a total of 200 g has been added to the cart. 7) Graph the results. An appropriate way to assess the students' ability to use higher-order thinking skills to draw conclusions based on these experimental data would be to have the students A) predict the distance the cart would travel if 250 g were added. B) determine how much farther the cart traveled with 200 g than with 100 g. C) convert the measurements into other units within the International System of Units (metric system). D) describe what the graph would look like if the divisions on the vertical axis were spaced farther apart.

A

A swarm of grasshoppers begins to eat the wheat in a field. Birds descend on the field and eat many of the grasshoppers, and snakes eat some of the birds. In the food chain described, which organism or group of organisms provides the greatest amount of energy? A) wheat B) grasshoppers C) birds D) snakes

A

An elementary teacher has ordered frog eggs from a biological supply company. the teacher means to put the eggs into an aquarium in the classroom, where the students can observe them over a period of time. As the students observe the transformation of eggs into tadpoles and then into frogs, one of the students asks, 'why do tadpoles have tails, but frogs don't?' The question would most likely provide an opportunity for the teacher to discuss which of the following topics? A) an organism's adaptive traits that allow it to survive in an environment B) the role of dominant and recessive traits in development C) the importance of the role of mutations in an organism's survival D) an organism's physiological response to environmental stressors

A

Mr. Samoa will be helping sixth graders understand the characteristics of limited and unlimited governments. Which of the following concepts should the students be familiar with before he begins the lesson? A) the purpose of laws B) the concept of sovereign authority C) the advantage of free markets D) the naturalization of citizens

A

Ms. Aguirre has several English-language learners (ELLs) in her class. To provide her ELL students with additional support, Ms. Aguirre often incorporates body movement into her verbal interaction with her students by clapping the syllables of words in simple sentences. Her approach focuses primarily on which of the following skills? A) phonological awareness B) pragmatics C) phonics D) syntax

A

Ms. Ruiz is teaching her students about igneous, sedimentary, and metamorphic rocks. Which of the following activities would foster critical thinking in her students? A) students classifying rock samples as igneous, metamorphic or sedimentary B) students watching a video that details the characteristics of igneous, sedimentary and metamorphic rocks C) students researching and then listing the characteristics of igneous, sedimentary, and metamorphic rocks. D) students labeling diagrams of igneous, sedimentary, and metamorphic rocks using their textbook

A

Teaching a lesson on the properties of sound, a teacher talks about the sound of a passenger jet plane in flight, observing that the sound seems to come from a point several plane-lengths behind the jet itself. This example best demonstrates which of the following about the speed of sound? A) it is significantly slower than the speed of light B) it is significantly slower than a jet's cruising speed C) It stays the same regardless of the atmospheric conditions D) It is slower for low-pitched sounds

A

Use the table below to answer the question. Milligram (mg) 1000mg = 1g Gram (g) base unit Kilogram (kg) 1000 g = 1 kg Which of the following is equivalent to 2g? A) 2000 mg B) 20000 mg C) 200000 mg D) 2000000 mg

A

What unit of measure is used when finding the girth, G, of a box, which is related to the distances around a box, by the formula G = 2(w+h)? A) cm B) cm^2 C) cm^3 D) cm^4

A

Which of the following energy transformations occurs when gasoline is ignited in the cylinder of a car engine? A) chemical energy is converted to thermal energy B) thermal energy is converted to chemical energy C) kinetic energy is converted to thermal energy D) electrical energy is converted to kinetic energy

A

Which TWO of the following will most likely increase labor productivity? a) Competition b) Division of labor c) An increase in the supply of goods d) An increase in the cost of physical capital

A,B

Which THREE of the following statements most accurately describes the impact that learning conventional spelling has on writing development? A) It encourages students to expand their writing vocabulary. B) It supports student's ability to communicate ideas effectively. C) It precedes student's understanding of the rules of grammar and punctuation. D) It allows students to focus on more complex aspects of writing, such as organization and audience. A, B, D A, C, D B, C, D A, B, C

A,B,D

Which of the following is the most important function of stomata, the small openings found primarily on the surfaces of plant leaves? Allowing pollinators to access the reproductive parts of the plant Allowing gases to be exchanged between the plant and the atmosphere Allowing water to be absorbed from the surface of leaves Allowing sunlight to enter the cells of the leaves

Allowing gases to be exchanged between the plant and the atmosphere

Which of the following statements about the stages of writing development is best supported by research? Students taught writing conventions in isolation show greater mastery than those taught writing skills in conjunction with reading skills. Although most students progress through predictable stages of writing development, the rate of skill development can vary. In general, students acquire handwriting skills through literacy exposure and do not need explicit instruction in letter formation. Most students learn to form letter symbols before comprehending that written symbols are used to convey meaning.

Although most students progress through predictable stages of writing development, the rate of skill development can vary.

Of the following, which best accounts for the phases of the Moon as seen from Earth? The motion of the Moon around Earth The orbit of an asteroid around the Moon The changing position of clouds in Earth's atmosphere The shadow of Earth passing over the surface of the Moon

The motion of the Moon around Earth

Which of the following color schemes uses colors that are adjacent to one another on the color wheel and contain a common color (e.g., blue-green, green, and yellow-green)? Analogous Complementary Monochromatic Split complementary

Analogous

A teacher plays Pachelbel's "Canon in D" while students use silent movement to express their reaction to the music. Afterward, the teacher divides the students into groups of four, instructing them to compose group poems about the experience, with each student contributing a line or two. Which of the following is the primary purpose for which the teacher is using movement and sound? As an introduction to dance As an introduction to improvisation As a catalyst for creative writing As a catalyst for conflict resolution

As a catalyst for creative writing

A fourth-grade student is creating an article for a fictional newspaper that discusses a specific difference between Sam Houston and Mirabeau B. Lamar. Which of the following headlines would be most appropriate for the student to use? Heroes of the Texas Revolution Attitudes Toward American Indians Presidents of the Republic of Texas Texans Urge for British Colonial Procurement

Attitudes Toward American Indians

A Fourth-grade teacher has each student choose a novel and then places the students into small groups based on their book choice. Each group meets periodically. While the teacher facilitates each group's meetings, the students decide the reading selections and discussion topics. The instructional practice outlined can be best described as A) guided reading B) literature circles C) shared reading D) choral reading

B

A first-grade teacher meets with her students before their first visit to the library. She shares with students the following routine. "Open the book to the middle. Read the page to yourself. Hold up one finger for each word you don't know how to pronounce. If you get to five fingers, the book is too hard. Pick a different book." The main purpose of the teachers' instruction is to guide the students to A) locate books in the library by author and by topic B) select books at an independent reading level C) read fluently with few errors in pronunciation D) practicing decoding new and unfamiliar words

B

A fourth-grade teacher uses readers' notebooks weekly in her class. Students write in their notebooks, and the teacher responds to each student individually, using the notebooks as a tool for ongoing communication. This week the teacher provides a variety of picture books and chapter books on a fourth-grade reading level or below and allows students to select one. After reading their books silently, students receive the following instructions from the teacher: Write a letter to me in your reader's notebook, and answer the following two questions. 1)What do you think the author's main message to the reader is in the book you read and why? 2) Write a 1-3 sentence quotation from the book that best describes the character you like most. Which of the following is the best strategy for adapting the activity for a student with dyslexia? A) requesting that the student go to the library and choose a book on tape to better understand the story B) having the student dictate his letter to a scribe and then respond to the letter orally in a one-on-one conference. C) helping the student in choosing a book below his instructional reading level to maximize comprehension D) establishing a time limit to ensure the student completes the letter in class and receives the appropriate assistance.

B

A social studies teacher is beginning a unit on the colonization of North America. As an introduction to the unit, the students will read a selection of journal entries from a crew member who accompanied Christopher Columbus on his first voyage to the New World. The journal entries are examples of A) expository text B) narrative text C) an autobiography D) historical fiction

B

An elementary teacher has ordered frog eggs from a biological supply company. the teacher means to put the eggs into an aquarium in the classroom, where the students can observe them over a period of time. Before introducing the frog eggs into the aquarium, the teacher is ethically obligated to A) be certain that the aquarium is completely sterilized and filled with distilled water B) provide all of the basic requirements for the survival and development of the eggs C) make preparations to release the frogs into a local pond as soon as they have matured D) introduce different organisms into the aquarium to simulate a natural pond ecosystem.

B

During the morning message, a kindergarten teacher produces the /t/ sound and asks the students 'Who can show me the letter in the morning message that makes that sound?' A student then uses a pointer to identify the letter that corresponds with that sound. Which of the following concepts is the teacher primarily addressing? A) phonemic awareness B) alphabetic principle C) fluency D) schema

B

In a classroom demonstration, a science teacher pours hot water into a beaker and covers it. After a few minutes, she removes the lid and asks the students to examine it. The students observe that there are water droplets on the lid. Which of the following processes produced what the students observed? A) transpiration, followed by condensation B) Evaporation, followed by condensation C) transpiration, followed by precipitation D) precipitation followed by evaporation

B

In a lesson about resource use, a teacher is contrasting the use of fossil fuels with the use of biofuels. Which of the following best characterizes how biofuels are different from fossil fuels? A) biofuel combustion does not release carbon dioxide B) biofuels are renewable resources C) the energy in biofuels originate from the Sun D) the reliance on fossil fuels decreases the availability of fossil fuels.

B

In a pictorial model, which contains 100 squares, seven of the ten columns are shaded, and the eighth column has two shaded squares. If each unit square in the 100 square model represents 0.1, then which of the following is the decimal numeric representation of the shaded area? A) 72 B) 7.2 C) 0.72 D) 0.072

B

Mr. Alvarez has been reviewing the letters of the alphabet with his first graders. to help reinforce their recognition of the shapes of the letters, he plans a physical-education activity for the students. Mr. Alvarez divides the class into pairs. He then says the name of a letter, and each pair of students must think of a way to make the shape of the letter with their bodies. the activity is most likely to promote students' physical development in which of the following areas? A) eye-foot coordination B) body awareness C) eye-hand coordination D) tactile awareness

B

Renee, a pre-kindergartener, shows her teacher a picture she has drawn of her puppy. She tells the teacher that the print on the page says 'This is my puppy, Oscar,' but the print actually reads ENREE. The writing demonstrates that she has an understanding about which of the following concepts of print? A) words are read from left to right B) Print carries meaning C) letters correspond with sounds D) sentences are composed of words

B

The human skeletal system consists of bones, cartilage, ligaments, and tendons. Which of the following sentences correctly states one of the functions of the human skeletal system? A) it provides the energy to move the body B) it provides a rigid framework that supports and protects many organs of the body C) it stores oxygen for use by the body during times of strenuous exertion D) it provides a network of channels for the movement of blood throughout the body

B

The students in Mr. Grant's third-grade class earn tickets for positive behaviors and use them to bid on prizes in a class auction at the end of each week. One Friday Mr. Grant went to his prize box and pulled out only one object, a popular multicolored pen. he explained to the students that it was the only prize he had and began the bidding process. The bidding reached fifteen tickets. He then pulled out twenty more of the same pens and began the bidding again, but no one would bid more than one ticket. The application of which economic concept can be demonstrated by the impact of the twenty additional pens on the bidding price? A) opportunity cost B) supply and demand C) price floor D) comparative advantage

B

Third grade students have been identifying planets in the solar system and their position in relation to the Sun. After completion of the unit, which of the following culminating activities would best assess the students' learning? A) students replicate a unit experiment that uses the scientific method to demonstrate the effects of gravity B) students create a display board of unit concepts to present in a science fair C) students complete a self-evaluation of their learning of unit material by using a rubric D) students present an oral report about a planet that incorporates the use of both print and nonprint resources

B

Which of the following did Lyndon B. Johnson, the thirty-sixth president of the United States, accomplish during his tenure in Congress (1937 to 1961)? A) becoming a leading expert on international policy through his work on the Senate Foreign Relations Committee B) mustering congressional support for rural electrification, the space program, and civil rights legislation C) coordinating nationwide efforts to reduce election fraud D) establishing new regulatory policies for the radio and television industries

B

Which of the following technologies had the greatest impact on the environment and geography of agriculture in the U.S. Great Plains? A) the cotton gin B) the steel plow C) the telegraph D) the prairie schooner

B

Which TWO of the following statements best reflect the relationship between oral-language development and reading comprehension? A) Development of reading comprehension typically precedes development of listening comprehension B) Having multiple, varied opportunities to interact with oral language will positively affect reading comprehension C) Weakness in phonological awareness is generally a predictor of future weakness in reading comprehension D) Early emphasis on the development of oral-spelling skills has a positive development of reading comprehension skills. A,B A,D B,C C,D

B,C

A first grade teacher who is working with a group of beginning readers gives each student a set of word cards. On each card is printed a word that the students have already learned to read (e.g. he, she , sees, loves, has, the, a, dog, cat, and pail). The teacher shows the students how to arrange the cards to create a statement (e.g. She sees the cat.). Students then create their own statements and read them aloud. One goal of the activity is to promote students' reading development by reinforcing word-recognition skills. In addition, the activity can be expected to promote students' writing development by A) helping them learn to view writing as a useful form of communication B) promoting the recognition of similarities and differences between written and oral language C) building their understanding of basic syntactic structures D) helping develop their understanding of the value of writing conventions (e.g. capitalization, punctuation).

C

A pre-kindergarten teacher could best promote the development of students' listening skills by A) pausing occasionally when speaking to ask individual students to repeat what was just said B) using attentive listening behavior when students are speaking C) integrating specific listening activities as a routine element in the daily schedule D) frequently reminding students to think about what they are hearing

C

A second-grade teacher gives each student a set of hand bells. She asks students to sing the song Twinkle Twinkle, Little Star and shake the bells to the beat of the music. The main purpose of this activity is to instruct students about the A) pitch of the bells B) timbre of the bells C) meter of the music D) intonation of the voices

C

A second-grade teacher is focusing on the concept that change occurs in systems. Which of the following activities presents that concept using the scientific inquiry method? A) comparing and contrasting two seasons by using a Venn diagram B) watching a video explaining the effects of Earth's revolution around the Sun C) making and testing predictions of how long it would take an ice cube to melt on a sunny day D) reading a nonfiction book about how farmers use scientific data in the production of crops

C

A teacher has each of his kindergarten students flip a coin twenty times while keeping track of which side of the coin lands up - heads or tails. This is an example of an activity that will help the students develop a beginning understanding of A) number patterns and functional relationships B) real numbers and laws of order of operations C) data collection and probability D) fraction sense and part-to-whole ratios

C

A teacher is using a model of the solar system to demonstrate the seasons. To give an accurate demonstration, the teacher should focus on which of the following features? A) the position of Earth's orbit compared to the orbits of neighboring planets B) Earth's changing distance from the Sun as it goes through its orbit C) The tilt of Earth's rotational axis D) The velocity of Earth's rotation on its axis

C

In a pictorial model, which contains 100 squares, seven of the ten columns are shaded, and the eighth column has two shaded squares. What fractional part of the model is shaded? A) 72/1 B) 72/10 C) 72/100 D) 72/1000

C

Mr. Kaper's pre-kindergarten class has been studying the neighborhood surrounding their school and have taken many walking field trips to businesses and restaurants. Mr. Kaper posts pictures from each of the places they visited. Which of the following activities would provide the best opportunity for the students to compare and interpret information using these visual images? A) The class will create a class book describing the places they visited. B) Students will draw pictures of their favorite place the class visited C) The class will make a picture-graph categorizing each place they visited D) Students will complete an independent project depicting their homes

C

Mr. Lawrenson wants to help his kindergarten students learn to express themselves through movement by applying different interpretive movements to a poem they are reading in class. To best help students complete the activity, Mr. Lawrenson should first A) demonstrate for students which movements to use with the poem B) have students perform a movement of their choice when they hear a rhyming word int he poem. C) brainstorm with students types of movements that illustrate the poem D) give the students a list of movements that are appropriate for the poem

C

Ms. Gonzales has her third-grade students read the following sentence from the science text to themselves. 'After the volcano erupted, lava flowed down into the forest, destroying the trees and vegetation.' One of the students asks Ms. Gonzales what the word 'erupted' means. Which of the following actions by Ms. Gonzales would best foster students' independent use of reading strategies? A) Providing the students with sentences that use the word correctly and sentences that use the word incorrectly B) giving the students a quick and sample definition of the word as it relates to the text of the reading C) modeling the process of using the surrounding text to determine the meaning of the word D) instructing the students to reread the sentence until they discover the meaning of the word on their own.

C

Ms. Hall, a first-grade teacher, has been teaching a unit on safety with her class. She has already taught several lessons on how to stay safe, prevent accidents and respond to emergencies. In order to best assess her students' knowledge, she should A) show a video on safety to the class and have them write a summary B) invite civil servant guest speakers to discuss safety with the class C) assign small groups to create a written action plan for a specific emergency situation D) assign small groups to role-play their reactions to various emergency situations

C

Ms. Moore is working with her sixth-grade science class to spot simple machines in daily surroundings. Which of the following everyday objects would be the best example of a lever? A) a ramp B) a skateboard C) a hammer D) a flag

C

Propane C3H8 is a common fuel used in home barbecue grills. When propane burns in the air, it produces which of the following gases? A) helium B) oxygen C) carbon dioxide D) nitrogen

C

Which of the following best explains why a citizen who yells 'fire' in a crowded room where there is no fire is not protected by the First Amendment to the United States Constitution? A) Individual rights are only protected when individuals state facts rather than opinions B) the first amendment protects individual rights at the local level but not at the federal level C) no right is absolute, especially when weighing an individual's rights against the rights of a larger group D) the first amendment only protects written documents and not speech

C

Which of the following is one of the layers of Earth's atmosphere? A) corona B) nebula C) stratosphere D) biosphere

C

Which of the following would be a pre-kindergarten teacher's best strategy for helping students understand the concept of one-to-one correspondence? A) Have the students pretend to purchase objects using play money in denominations of pennies, dimes and dollars B) Provide the students with sets of similar objects to arrange in order of size C) Have the students take turns setting the table at snack time, placing a cup or napkin in front of each chair D) Provide the students with objects to compare in terms of features such as color and length.

C

Which TWO of the following multisensory activities most directly support kindergarten students' understanding of the relationship between sounds and letters? A) Drawing letters in the air while singing the alphabet song. B) Acting like an animal whose name starts with a given sound. C) Shaping their bodies into the letter that matches a given sound D)Making a letter's sound while using their finger to trace the letter in sand. A, B A, D B, C C, D

C,D

Which of the following statements correctly explains part of the system of checks and balances in the United States federal government? International treaties must be written in Congress and signed into law by the president. The president has the power to appoint and impeach Supreme Court justices. The judicial branch has the power to veto proposed legislation drafted by Congress. Congress can override a presidential veto on legislation with a two-thirds vote from both houses.

Congress can override a presidential veto on legislation with a two-thirds vote from both houses.

Ms. Wingate is teaching a lesson on patterns. She provides her students with cubes of various colors and instructs them to create a pattern of their own. After the students each create a pattern, they will take turns presenting and explaining their patterns. Which of the following theories of learning best matches the activity Ms. Wingate is using with her students? Social learning theory Constructivist learning theory Sociocultural learning theory Behaviorism learning theory

Constructivist learning theory - students are active learners, that learn by doing and engaging in metacognition.

While reading aloud from a nonfiction text, a teacher pauses and says, "I'm not familiar with the word 'phenomenon.' I'm not sure what it means, so I'm going to look for some clues in the surrounding sentences to help me figure it out." Which of the following vocabulary-development strategies is the teacher modeling? Contextual analysis Semantic feature analysis Morphemic analysis Word hunt for patterns

Contextual analysis

Which of the following activities best promotes student vocabulary development after a read-aloud? Creating pictures to illustrate the meaning of the words Using a dictionary to write the definitions of the words Earning individual rewards for using the words in class Writing the words during daily guided writing instruction

Creating pictures to illustrate the meaning of the words

A fourth-grade teacher uses readers' notebooks weekly in her class. Students write in their notebooks, and the teacher responds to each student individually, using the notebooks as a tool for ongoing communication. This week the teacher provides a variety of picture books and chapter books on a fourth-grade reading level or below and allows students to select one. After reading their books silently, students receive the following instructions from the teacher: Write a letter to me in your reader's notebook, and answer the following two questions. 1)What do you think the author's main message to the reader is in the book you read and why? 2) Write a 1-3 sentence quotation from the book that best describes the character you like most. Explain why this character is your favorite and how the quote reflects the character. The main purpose of the second writing topic is to increase students' A) comprehension of the main points of the book B) Use of imagination and creativity C) interest in reading different genres of books D) ability to support their textual interpretations

D

A group of fourth-grade students wished to test a hypothesis that adding mass to a cart rolled down a ramp would increase the distance the cart would travel along the floor. To do this, they designed and carried out the following experiment using a toy cart with a mass of 200 g, a small ramp and several 25 g masses. 1) Make a line across the ramp near the top. 2) Line up the front wheels of the cart with the line. 3) Let go of the cart without pushing it. 4) When the cart stops, measure the distance from the bottom of the ramp to the place where the front wheels stopped, write down this distance. 5) Add a 25 g mass to the cart. 6) Repeat until a total of 200 g has been added to the cart. 7) Graph the results. The students were concerned about the measurement they obtained when 75 g were added to the cart because it did not seem to fit into the pattern created by the other results. Now they would like to repeat the investigation to determine whether this distance is correct. To improve their investigation and to get more reliable results, the students' best modification would be to A) measure the total distance the cart traveled from the top of the ramp to where it stopped. B) have two students measure independently the distance the cart traveled. C) change the height on the ramp at which the cart was released. D) release the cart several times at each mass and take the average distance traveled.

D

A teacher is working with a group of first-grade students on exploring the concept of ten. Which of the following activities would be most effective in helping the students grasp the concept of ten? A) coloring in the even numbers on a hundred chart B) estimating the number of chocolate chips in a bag C) writing the numerals for one to ten on dry-erase boards D) putting one bean in each square of a grid containing ten squares.

D

Mr. Morrison, a pre-K teacher, has his students observe caterpillars developing into monarch butterflies over the course of a few weeks. The students are excited to see each butterfly emerge from a chrysalis. In addition to having this experience serve as a science lesson, Mr. Morrison could use the butterflies as a stimulus for artistic expression appropriate to this age group by A) showing the students how to make origami butterflies by carefully folding and twisting thin paper B) asking each student to draw a series of pictures that accurately show the stages of a monarch butterfly's life cycle C) asking each student to use the patterns on the butterflies' wings as the basis for an intricate, multimedia collage D) encouraging the students to observe the butterflies closely and make butterfly-inspired pictures with materials in the art center

D

Mr. Silva's first grade students make a chart of their predictions about a story prior to reading a story. As they read the story, they refer to the chart to confirm or change their predictions. The primary instructional purpose for the activity is to help students A) understand how to use a story map to organize thoughts B) develop a richer and more extensive vocabulary C) recognize that writing is connected to reading D) make inferences to aid in comprehension

D

The day after a cold front passes, an area is most likely to experience A) severe weather B) cloud cover C) drought conditions D) lower temperatures

D

To best assess a student's accuracy and rate of reading, a teacher should have a student A) read a passage silently for one minute and then write a summary of it B) read out loud for one minute from a list of words of varying difficulty while the teacher records miscues C) read a passage with words omitted out loud for one minute and then fill in the blanks with appropriate words D) read a passage out loud for one minute while the teacher records miscues

D

Which of the following civilizations is most closely associated with the systematic study of geometry and the development of mathematical proofs? A) Egyptian B) Persian C) Roman D) Greek

D

Which of the following is an example of a purely instinctive, inherited trait? A) a dog fetches a stick B) a dolphin jumps through a hoop C) a pride of lions hunts for prey D) a snake slithers across the ground

D

While teaching sixth-grade students new abstract science concepts, a teacher plans to use questioning strategies to help his students construct and monitor teaching. Which of the following techniques would best promote students' understanding? A) students self-assess their understanding of the concepts by answering a set of teacher-generated multiple-choice questions B) students answer questions that appear after a textbook chapter covering the concepts C) students respond to teacher-prompted discussion questions about the concepts from the most complex levels of Bloom's taxonomy D) students generate their own questions about the concepts before beginning instruction

D

Which of the following statements best aligns with current research on teaching decoding- and word-analysis skills to students? Guessing at unfamiliar words should be modeled as a successful decoding strategy. Decoding should be taught in relation to a student's stage of reading development. Students should be taught to decode print before they attempt to write words. Phonics instruction should precede phonological-awareness instruction.

Decoding should be taught in relation to a student's stage of reading development.

Which of the following student activities will most effectively help students to think critically about the role of plants in an ecosystem? Designing a functioning terrarium that includes plants and animals Labeling a picture of the living and nonliving factors of an area that includes plants Searching for articles about the life cycles of plants on the Internet Comparing pictures of plants in various biomes

Designing a functioning terrarium that includes plants and animals

While previewing a text, fifth-grade students identify the signal words "because" and "if... then" in the section's text. Which of the following comprehension skills are the students primarily utilizing? Examining text features Recognizing genres of texts Determining text structures Identifying purposes for reading

Determining text structures

Which of the following activities will best allow a teacher to demonstrate socioeconomic diversity for students in a math class? Graphing the income of the teachers at the school to compare their finances Encouraging students to adjust their lunch budgets to participate in the school fundraisers Comparing the weekly allowance given to students by their parents Discussing in groups how to plan budgets for various income amounts

Discussing in groups how to plan budgets for various income amounts

A sixth-grade teacher plans to assign a research project about Africa. Each student will be using various print resources as well as the Internet to research aspects of culture, society and government in different parts of Africa. Students will create artistic projects as well as written products, and each student will present his or her findings to the class. The teacher sketches out her goals for the project. Goals: 1. Student will identify characteristics of reliable Web sites. 2. Students will compare and contrast electronic and print media. 3. Parents will be aware of project goals and expectations. Which of the following best achieves goal 2 ? Examining features of a textbook, such as bold print and graphics that aid in comprehending online texts Reading a magazine article on a computer and tablet to determine which version is easier to read Reviewing a research article in comparison to a newspaper editorial to identify levels of formality Seeing a documentary film and reading online reviewer comments to gauge viewer understanding

Examining features of a textbook, such as bold print and graphics that aid in comprehending online texts

An instructor has demonstrated to students how different lighting instruments and colored gels can work to enhance the stage area. Which of the following theatrical terms most pertains to lighting and lighting design? Arena Drop Fresnel Apron

Fresnel

Which of the following methods for teaching students to use citations in research will produce the highest level of student engagement? Using a slide show to display various ways of citing research Having students work in pairs to interview and cite one another Leading a whole-class discussion about the significant points of proper citation Watching a video of a researcher discussing the importance of proper citation

Having students work in pairs to interview and cite one another

A student asks the teacher for assistance reading a multisyllabic word. Which of the following strategies will best support the student in learning to decode complex words independently? Using a dictionary to find the meaning and pronunciation of the word Listening as the teacher demonstrates how to pronounce the word correctly Pronouncing each individual letter sound, then blending the sounds together Identifying syllables or chunks and highlighting each differently

Identifying syllables or chunks and highlighting each differently

Which of the following is included in the Texas Constitution Bill of Rights? The universal right to vote The right to equal protection under the law The right to maintain local militias The right to conduct international commerce

The right to equal protection under the law

Which of the following actions is the first step to take when teaching a song by rote? The teacher sings the entire song, without accompaniment, while students listen. The teacher explains new vocabulary, then reads the lyrics to the students. The teacher and students sing the song together using recorded accompaniment. The teacher sings the song one line at a time, echoed by the students.

The teacher sings the entire song, without accompaniment, while students listen.

A sixth-grade teacher plans to assign a research project about Africa. Each student will be using various print resources as well as the Internet to research aspects of culture, society and government in different parts of Africa. Students will create artistic projects as well as written products, and each student will present his or her findings to the class. The teacher sketches out her goals for the project. Goals: 1. Student will identify characteristics of reliable Web sites. 2. Students will compare and contrast electronic and print media. 3. Parents will be aware of project goals and expectations. Which of the following best achieves goal 3 ? Asking parents to proofread the written product and sign off on it before the due date Creating a list of possible goals and having parents and students agree to five goals Inviting students and parents to a project orientation night to view samples from previous students Encouraging older students with prior project experience to come in and help current students

Inviting students and parents to a project orientation night to view samples from previous students

A teacher wants to provide tips for parents to use at home to support students' development of reading fluency. Which of the following tips will best support the teacher's goal? Ensure your child is reading silently each evening. Listen to your child read aloud the same text multiple times. Have your child read aloud materials that are very challenging. Let your child observe you reading silently at home.

Listen to your child read aloud the same text multiple times.

Which of the following was the primary role of the Virginia House of Burgesses? Making laws Deciding legal cases Advising English colonial authorities Regulating trade with England

Making laws

A prekindergarten teacher creates a set of letter and number cards for students to sort. Which of the following visual supports would best enhance students' ability to distinguish letter cards from number cards? Using only the capital-letter symbols on each letter card Including an equal number of letter cards and number cards Limiting the number cards to single digits between one and nine Making the number symbols a different color than the letter symbols

Making the number symbols a different color than the letter symbols

Which of the following traditional folk songs promotes the heritage of Texas? A. "The Old Chisholm Trail" B. "Cotton-Eyed Joe" C. "Casey Jones" D. "Sweet Betsy from Pike"

Option A is correct because Texas ranchers used the Chisholm trail to drive cattle to the rail yards in Kansas.

During a unit on fiber art, it would be most appropriate for students to learn how to use a A. handloom. B. press mold. C. palette knife. D. lathe chisel.

Option A is correct because a handloom is a tool for weaving, a common kind of fiber art.

A type of song in which a second voice exactly imitates a leading voice at different places in the melody is a A. Round B. Partner song C. Lullaby D. Game song

Option A is correct because a round is a song in which a leader is imitated exactly at one or more certain points within the song.

A class has just completed a unit on the anatomy of a flower. Which of the following is the most effective way for a teacher to assess the students' understanding of the concept? A. Having the students dissect a flower and label the parts B. Having the students write a paragraph about the pollination of flowers C. Having the students write the definitions of the different parts of the flowers D. Having the students plant seeds and measure the time it takes for the plants to flower

Option A is correct because dissecting a flower and labeling the parts demands that the students make the connection between the appearance and location of the parts of an actual flower to descriptions and two-dimensional images of a flower.

Students are working independently to solve the equation - + =- 4 ? 10 . The teacher says the following to help them understand the problem. "If you owe somebody $4, you have a negative $4 balance with that person. If you borrow more money from the person, you will owe more and have a more negative balance with that person." After speaking with several students, the teacher finds that some of them are still having trouble with the concept of negative numbers. As a result, the teacher then reteaches the concept using a number line. Which of the following types of assessments has the teacher used? A. Formative B. Summative C. Formal D. Criterion

Option A is correct because formative assessment involves teachers' adjusting their instruction based on their assessment of students. The teacher in the scenario has conversations with the students and teaches again based on what is observed.

Which of the following is the adaptation that will best aid plant survival in an arid environment? A. Waxy leaves B. Aerial roots C. Broad leaves D. Green flowers

Option A is correct because having a waxy surface decreases water loss from leaves, aiding plant survival in an arid environment.

Fifth-grade students are preparing to read a short story about two children's visit to a New York City zoo. Which of the following activities is the best way to introduce the text to promote reading comprehension? A. Completing a KWL chart B. Listening to the story on audio C. Writing about their own experiences in New York City D. Identifying New York City on a map

Option A is correct because having students complete a KWL chart before and after reading the text helps build and activate background knowledge, which promotes reading comprehension.

Which of the following activities is most effective in helping kindergarten students understand measurement of the lengths of small objects, such as pencils or cups? A. Placing interlocking cubes next to the objects and counting the cubes. B. Cutting sheets of construction paper so that they are the same dimensions as the objects. C. Listening to the teacher explain how to line up a ruler next to the objects and mark their lengths. D. Watching the teacher demonstrate how to estimate the lengths of the objects using a child's hand or shoe.

Option A is correct because having the students count the number of interlocking cubes and placing the cubes next to each item provides a visual for the student to use in determining length.

The two most abundant gases in Earth's atmosphere are A. oxygen and nitrogen. B. oxygen and carbon dioxide. C. oxygen and water vapor. D. oxygen and hydrogen.

Option A is correct because in a sample of dry air near Earth's surface, approximately 99% is oxygen (about 21%) and nitrogen (78%).

During a back-to-school night, an elementary teacher discusses the use of media in the classroom and at home. Which of the following suggestions is best for the teacher to make to parents regarding students' television viewing and Internet use at home? A. Viewing media with their children and discussing what they see and hear B. Preventing children from having any media exposure at home, since most of it will be of little academic value to children C. Trusting children to make their own choices about media but asking them to discuss these choices with the parents D. Requiring children to watch certain educational programs recommended by the teacher before they watch any other programs

Option A is correct because in this way, parents are helping their children develop their media literacy skills instead of merely dictating what children may or may not watch or listen to.

A kindergarten teacher is creating a unit on apples to focus on the letter "A." Which of the following activities best contributes to students' understanding of the alphabetic principle? A. Cutting out pictures of objects that begin with "A" and labeling them in a book B. Creating a collage to be hung in the classroom of pictures of objects that begin with "A" C. Brainstorming words that begin with the same first letter as each student's name D. Inventing a song or rhyme to learn how to spell the word "apple"

Option A is correct because it requires students to identify pictures of objects that begin with "A" and to label them.

Which of the following is the relationship in which organism A benefits from living on or in the body of organism B but provides no benefit to organism B? A. Parasitism B. Predation C. Competition D. Mutualism

Option A is correct because parasitism is a relationship between organisms in which one organism benefits from living on or in the body of another organism, usually at the expense of the other organism.

In a meeting with a teacher, a parent asks how to encourage reading and improve reading skills for children at home. Which of the following is the most effective recommendation for the teacher to make? A. Reading books aloud with the child at home B. Allowing the child to see adults read at home C. Providing costumes and props for the child to act out stories at home D. Buying books on audio for the child to hear at home

Option A is correct because research shows that reading aloud with children is the best way to promote childhood literacy.

Which of the following activities is most effective in developing reading fluency skills in first-grade students? A. Rereading stories that were used during guided reading B. Participating in literature discussion groups in literacy centers C. Reading books that are at a level just above their independent reading level D. Completing a graphic organizer after reading a book the student has chosen

Option A is correct because students acquire reading fluency and comprehension from books read during guided reading lessons with their teacher. Repeated readings of familiar books improve reading fluency.

During a lesson on hitting a ball placed on a tee, a third-grade student is swinging the bat down on the ball from above rather than swinging parallel to the ground. Which of the following is most appropriate for the teacher to use first to help the student improve batting form? A. Physically guiding the swing to give the student the experience of proper technique B. Having the student watch other students perform the skill correctly and pointing out the other students' strengths C. Reminding the student to swing the bat in a plane that is parallel to the ground rather than in a vertical arc D. Using the bat to strike the ball from above, then from the side, and asking the student to watch where the ball goes each time

Option A is correct because students in a physical education setting generally learn best by doing.

A fourth-grade teacher has students read about famous figures from Texas history and then takes the class to see a play about the life and times of Davy Crockett. Taking the class to see the play will best accomplish which of the following objectives? A. Enhancing lessons in social studies through a culturally relevant theatrical event B. Creating critical consumers of theatre arts through an evaluation of historical fiction C. Having students use social studies skills to compare and contrast primary and secondary sources D. Having students recognize the influence of theatre on historical events

Option A is correct because students will have an opportunity to enrich their studies through a relevant theatrical experience. Option B is incorrect because the purpose is not to evaluate the performance; the students have not been instructed in how to evaluate or why they should do so. Option C is incorrect because there is no evidence that any primary sources (artifacts) were included in the students study or the play. Option D is incorrect because taking students to see the play would illustrate the influence of historical events on theatre, not the reverse.

Which of the following was the primary purpose of adding the Bill of Rights to the United States Constitution? A. Creating explicit limits on the power of the federal government B. Granting the power of judicial review to the Supreme Court C. Ceding sovereignty to the individual states D. Amending errors in the Constitution

Option A is correct because the Bill of Rights was adopted to address the fears of those who thought that the federal government's powers under the Constitution were too broad.

Which of the following was a major long-term effect of the westward expansion of the United States in the 1800s? A. The growth of United States agricultural and mineral exports B. An increase in tensions between the United States and Great Britain C. A shift in United States foreign policy toward isolationism D. The establishment of peaceful relationships with American Indian groups

Option A is correct because the acquisition of large tracts of land during United States westward expansion led to an extensive increase in the farming, ranching, and mining industries. The lands of the Midwest attracted farmers, miners, and lumber barons seeking rich natural resources.

A first-grade teacher is having students march to the sound of a drum. Which of the following instructions is most appropriate for teaching the movement concepts of spatial awareness and relationship awareness? A. Follow the person in front of you, but keep a comfortable distance between you. B. Stomp your foot on the fourth beat of each measure, and turn to your right. C. March faster as the drumbeat gets faster, but freeze when the drumbeat stops. D. Time your steps so that your foot touches the ground at the same time that the drumbeat occurs.

Option A is correct because the direction to follow the person in front but keep comfortable distance is focused on the movement concept of spatial awareness. Spatial awareness is the ability to understand the position of objects in relation to each other and to oneself. It includes understanding the relationship of objects when there is a change in position. Options B, C and D are incorrect because the directions focus on how the body moves, not where the body moves.

Texas rice farmers depend on water from the Colorado River to irrigate their crops. In times of severe drought, less water is released downstream, which sometimes results in inadequate irrigation and failed crops. To prevent such a loss, rice farmers sometimes plant alternative crops on their lands. This action is an example of A. adapting to environmental changes. B. adjusting to supply and demand. C. using nonrenewable resources to improve farming. D. modifying the environment to meet farming needs.

Option A is correct because the farmers are changing their choice of crop to adapt to the change in environment.

Second-grade students each place thin paper over an object they have found in the classroom and rub a crayon over the paper on part of the surface of the object. The activity can most effectively be used to introduce students to which of the following elements of art? A. Texture B. Space C. Line D. Shape

Option A is correct because the texture of each object will directly affect what the rubbings look like.

Most students in a kindergarten class can identify two words that rhyme. Which of the following phonemic skills is most appropriate for the teacher to address next? A. Naming the beginning sounds in words B. Blending the initial consonant sounds in words C. Pronouncing all the sounds in words with two or three phonemes D. Blending the final consonant sounds in words

Option A is correct because this is the next phase in phonemic awareness development.

Which of the following actions best represents the first step in the process students should use to interpret graphs and charts containing numerical information? A. Looking at the title, axes, headings, and legends to develop a sense of the content of the graph or chart B. Analyzing whether the type of graph or chart used is the best method of presenting the information given C. Determining the largest and smallest values represented on the graph or chart to get an impression of the data D. Comparing and contrasting the various areas of the graph or chart to determine high and low points of the data

Option A is correct because this is the previewing phase. Before analyzing any of the numerical data, students must be familiar with the content of the graph or chart.

Which of the following learning goals is most appropriate for a third-grade unit on money? A. Students will be able to determine the value of a collection of coins and bills. B. Students will be able to represent the value of a collection of coins as a fraction of a dollar. C. Students will be able to differentiate between money received as income and money received as gifts. D. Students will be able to solve problems involving money by performing operations on decimals to the hundredths place.

Option A is correct because this option correctly describes a Texas Essential Knowledge and Skill (TEK) for third grade math. A learning goal identifies what students will learn or be able to do as a result of instruction, not what they will be asked to do to demonstrate such learning.

Which of the following elements of writing is the best example of a major difference between written and spoken English? A. Sentence construction B. Pauses C. Figurative language D. Audience

Option A is sentence construction. Sentence construction refers to when writing the writer must follow rules and have a variety of sentence types. When speaking, individuals can sometimes use sentence fragments and run on sentences.

A first-grade class studies animals and categorizes them as either sea animals, pets, zoo animals, or farm animals. The teacher wants students to create a visual to display their categories. Which of the following will best display the information? A. A flow chart B. A concept map C. A line graph D. A bar graph

Option B is correct because a concept map will allow students to categorize the animals more easily.

Word problem: A paint store is having a sale, and for every gallon of paint a customer purchases, the customer will receive one additional gallon for free. Write an equation for p, the number of gallons of paint received, in terms of x, the number of gallons of paint purchased. A teacher asks students to solve the word problem shown. One student, John, says the answer is 2 + x = p. Which of the following activities will best help John recognize his misconception? A. Generating a model B. Creating a function table C. Using mental math D. Graphing the numbers

Option B is correct because a function table is a table of ordered pairs that follow a rule. The table will help the student identify the pattern.

Which of the following geometric solids has five faces, eight edges and five vertices? A. A pentagonal pyramid B. A rectangular pyramid C. An octagonal prism D. A triangular prism

Option B is correct because a rectangular pyramid has one rectangular base and four triangular faces, eight edges and five vertices.

Several students in a physical education class are having difficulty reaching the basketball goal with their shots, even while using a youth-sized basketball. Which of the following options is most appropriate for the teacher to introduce to increase the students' success? A. Allowing students to use a balloon instead of a basketball B. Lowering the height of the basket C. Raising the height of the basket D. Using a bouncy playground ball instead of a basketball

Option B is correct because by lowering the height of the basket, the teacher is decreasing the distance the students need to shoot and increasing the likelihood that they will succeed in making a basket.

A third-grade teacher assigns homework that requires students to list ten common words that can either stand alone or be combined with another word. The assignment primarily promotes the students' understanding of which of the following concepts? A. Analyzing sound-symbol relationships B. Making compound words C. Blending sounds in words D. Distinguishing root words

Option B is correct because compound words are words that can stand alone or can be combined.

As a demonstration during a unit on weather, a teacher adds food coloring to the bottom of a beaker of water being heated on a hot plate, and the students observe the movement of color in the water. Which of the following processes is illustrated by the movement of the color? A. Conduction B. Convection C. Evaporation D. Radiation Answer and Rationale

Option B is correct because convection is the process of heat transfer by currents in fluids. Convection plays an important role in both global and local weather phenomena. Option A is incorrect because conduction involves the transfer of heat by the exchange of kinetic energy between colliding particles. Option C is incorrect because evaporation is the process that occurs when liquid at the surface enters the air in the gas phase. Option D is incorrect because radiation is the process of energy transfer by electromagnetic waves.

Students are each given three identical small lumps of modeling clay and are asked to sculpt three different hearts from the lumps. For example, the hearts can be plump or slender, with varying proportions. The activity will best help students understand the art element of A. line. B. form. C. texture. D. light.

Option B is correct because form means the volume and shape of a threedimensional object. Varying the physical characteristics of the hearts made from the same amount of clay will create different forms.

A teacher notices that some students in the class have trouble interacting with peers during group assignments. Which of the following will best help students learn more appropriate oral language skills to use in group interactions? A. Assigning students to play a game in groups and then using a rubric to evaluate their interactions B. Rehearsing positive group collaboration by using a script and having students analyze productive conversations C. Requiring students to practice effective social interactions with their parents and siblings at home D. Videotaping a group while the members work together and allowing the class to view the video and then make suggestions for improving communication

Option B is correct because giving students a script provides them with a model of appropriate interaction that they can imitate and recall during future group assignments.

A common type of family abuse characterized by verbal put-downs and criticisms is known as A. abuse of power. B. emotional abuse. C. physical abuse. D. sexual abuse.

Option B is correct because hurtful language can cause emotional distress and is recognized as a form of abuse.

To best ensure active engagement for students working on an open-ended research question for science, a fifth-grade teacher should focus on providing which of the following? A. Explicit teaching B. Inquiry-based instruction C. At-home research projects D. Didactic questioning

Option B is correct because inquiry-based instruction ensures active engagement from students in the question, its research, and the development of a conclusion.

Which of the following is a responsibility of citizenship? A. Running for elected office B. Paying federal taxes C. Serving in the military D. Participating in elections

Option B is correct because it refers to an obligatory rather than a voluntary aspect of citizenship.

Which of the following stages of spelling development is primarily characterized by letter-sound correspondence? A. Semiphonetic B. Phonetic C. Transitional D. Conventional

Option B is correct because phonetic spelling, also known as invented spelling, requires students to use their knowledge of sounds in words and then assign corresponding letters.

A first-grade teacher plans to use the following passage from a nonfiction text as the introduction for a series of literacy lessons about tigers. As part of the unit, students will use the nonfiction excerpt and other research to create their own stories about a tiger. Passage: The tiger is a large cat found in Asia and parts of Russia. Most tigers are black, white and orange, although some are only white and black. They can run at speeds of 35 miles per hour, even though they are the largest species of cat. Tigers can weigh 400-675 pounds, and their bodies can be up to nine feet long. Is your body as long as a tiger? Tigers are endangered in the wild, because people often destroy their forest homes. In the United States, tigers can be seen in zoos and wildlife preserves. Have you ever seen a tiger? The following is the fictional story written by Jack: Sammy the baby tiger lives at a zoo in Texas. He like to play with his toy ball. He also like to play with his bruther and sistur. Which of the following text elements is most appropriate for the teacher to introduce in preparation for the writing assignment? A. Inclusion of illustrations B. Plot development C. Incorporation of facts D. Conflict resolution

Option B is correct because plot development is a characteristic of fiction text.

Which of the following forms of movement most clearly demonstrates basic skill in nonlocomotor body management? A. Hopping up and down B. Standing on a balance beam C. Leaping with variation in distance D. Throwing an object various distances

Option B is correct because standing on a balance beam is a form of movement that demonstrates the basic skill of nonlocomotor body management and balance.

When selecting vocabulary from magazine or newspaper articles used in instruction, which of the following methods is most effective? A. Selecting vocabulary based on the teacher's experience from previous years of instruction B. Asking students to identify words for further study when they read an article for the first time C. Using newspapers and magazines specifically made for school use, with bolded vocabulary terms D. Cross-referencing articles taught in class with district vocabulary lists to identify key words

Option B is correct because student selection of words for further study is the best way to ensure that the class is studying words that students will find useful and memorable.

Which of the following should students be able to do in order to begin improvising songs? A. Notate from dictation B. Sing in tune C. Read music D. Know the blues style

Option B is correct because students must sing in-tune before they can improvise tonally.

A fourth-grade teacher notices several students who are shy and reluctant to speak in class. What strategy would be most effective in motivating students' participation in class while enhancing their listening and speaking skills? A. Allowing students regular opportunities to read aloud in front of the class B. Giving students opportunities to share and listen to stories in a small-group setting C. Encouraging students to orally respond to higher-level thinking questions during class D. Permitting students to choose which groups of students they feel comfortable speaking to

Option B is correct because telling personal stories, listening to others' stories, and providing feedback and questions help build students' linguistic fluency, meaningful oral expression, confidence, receptive language skills, and listening comprehension skills. In addition, being in a small-group setting will put the students more at ease.

Which of the following had the greatest impact on Anglo settlement in Hispanic Texas? A. Native Americans encouraged trade with Anglos. B. Mexican land grants provided inexpensive properties in Texas. C. Government payments enticed settlers to inhabit American Indian territory. D. The prospect of religious freedom in Texas attracted Anglo settlement.

Option B is correct because the Mexican government approved land grants to encourage Anglos to move to Texas. Anglos could purchase land for much less in Texas than in the United States, which encouraged large groups of Anglos to move to Texas.

Which of the following phrases best describes the human digestive system? A. A group of similar organs that function independently B. A continuous tube with attached organs that performed different functions C. A group of similar organs connected by a network of blood vessels D. A system of organs that provide for food storage in the body

Option B is correct because the digestive system is a continuous tube that runs from the mouth to the anus and contains attached organs that perform different functions that aid in digestion and the breakdown of food.

A fifth-grade teacher writes the problem 56 * 12 on the board. Students begin to solve the problem mentally, and as each student finds a solution, he or she signals the teacher with a thumbs-up signal. When almost every student has given a thumbs-up signal, the teacher has the following dialogue with a student. Teacher: "Billy, what answer did you come up with?" Billy: "792." Teacher: "Great job, Billy! That is the correct answer. Raise your hand if you found 792 to be the product, like Billy." Almost every student in the class raised a hand. The teacher writes the next problem on the board. Which of the following instructional adjustments can the teacher make to best assess all of the students' understanding of multiplying two-digit numbers? A. Allowing students to write their answers on paper, then collecting the papers at the end of the lesson B. Asking multiple students to share and defend their solutions before acknowledging the correct answer C. Asking students who did not hold up their thumbs to share their answer and explain D. Having Billy work the problem out on the board in front of the class

Option B is correct because the discussion gives the teacher an opportunity to hear how students are solving the problem. It will also give students an opportunity to share before they know their answer is wrong.

After learning the theoretical probability of a two-sided coin landing on any one side, students work in groups to flip the coin several times and get the following results: 9 heads and 6 tails. Based on the scenario, which of the following observations made by students about probability is accurate? A. The theoretical probability of a coin's landing on heads, 0.5, is equal to the experimental probability obtained. B. The theoretical probability of a coin's landing on heads is less than the experimental probability obtained. C. The experimental probability of a coin's landing on heads, 0.6, is lower than expected. D. The experimental probability would have been more accurate if the students had decreased the number of trials.

Option B is correct because the experimental probability and the theoretical probability of the coin's landing on heads are respectively 0.6 and 0.5. The theoretical probability is thus less than the experimental probability.

Which of the following statements best describes the continuing role of familial oral storytelling traditions in many cultures around the world? A. Stories have become the primary means by which cultures interact with one another in an increasingly globalized world. B. Oral traditions serve as a means of transmitting knowledge of cultural practices and societal norms to others. C. New advances in technology have made oral traditions largely irrelevant in most cultures around the world. D. Religious groups utilize storytelling mainly to discredit the beliefs of other religions and cultural practices.

Option B is correct because the family unit uses storytelling to transmit knowledge of the culture and society in which it exists

Of the following activities, which best introduces to students the process skill of classification? A. Measuring the time it takes a marble to reach the bottom of an inclined plane when dropped from different heights B. Sorting a collection of objects into metals and nonmetals, then sorting the metals into magnetic and nonmagnetic C. Modeling the faces of the moon using a foam ball and light, then describing and drawing their observations D. Representing the stages of the life cycle of a butterfly using craft materials provided by the teacher

Option B is correct because the students are performing two consecutive binary classifications based on the physical properties of the objects.

A fourth-grade teacher is beginning a unit on energy and matter that contains vocabulary not regularly used by students. To best draw on students' experiences in a manner that helps them develop a deeper understanding of the unit's terms, the classroom teacher should A. have numerous dictionaries available for students. B. build a language-rich environment. C. ask students to choose a partner and define key words. D. encourage struggling students to use their science journals.

Option B is correct because vocabulary lessons are built on language-rich environments to support word learning.

When wood burns, which of the following energy transformations occurs? A. Nuclear energy into mechanical energy B. Chemical energy into heat energy C. Light energy into mechanical energy D. Electrical energy into heat energy

Option B is correct because when wood burns, chemical energy is converted to heat energy. Some of the chemical energy stored in the chemical bonds of the substances in wood is released as heat during the combustion process.

Which THREE of the following events most directly affected the economic development of Texas in the twentieth century? A. The attack on Pearl Harbor in 1941 B. The Arab oil embargo of 1973 C. The oil strike at Spindletop in 1901 D. The stock market crash of 1929 E. The Cuban missile crisis of 1962

Option B is correct because, although the embargo placed on oil products from the Middle East caused distress for the rest of the United States, the oilproducing state of Texas experienced an economic advantage because of price inflation. Option C is correct because the discovery of oil at the Spindletop salt dome near Beaumont, Texas in 1901 can be attributed to the birth of the modern petroleum industry in Texas. The oil strike opened new avenues of business for prospectors, oil refineries and manufacturers. Option D is correct because, although many Texans were skeptical that the stock market crash of 1929 would have serious effects on their state economy, they began to change their positions on seeking federal aid as conditions during the Great Depression worsened. New Deal programs such as the Civilian Conservation Corps (CCC) and the Works Progress Administration (WPA) benefited the state during its economic downturn by alleviating unemployment issues and building new infrastructure.

Which of the following events was a result of human interaction with the environment during the Spanish colonial era of Texas history? A. The construction of presidios near churches B. The conversion of American Indians to Christianity C. The building of acequias for the missions D. The creation of alcaldes for civil settlements

Option C is correct because acequias were ditches dug to direct river water toward the missions

Which of the following is the best example of a learned behavior in animals? A. A bird migrating in the spring B. An opossum hiding in grass when startled C. A chimpanzee using tools to access food D. A bear hibernating in winter

Option C is correct because chimpanzees learn to use tools by watching older chimpanzees use tools.

According to current research in the field of physical education, which of the following best promotes student participation in lifelong physical activity? A. Learning how to play popular sports and games B. Playing for a team that wins games regularly C. Acquiring basic skills needed to participate in a variety of activities D. Becoming familiar with common gymnasium equipment

Option C is correct because current research indicates that when students acquire the basic skills they need to participate in a variety of activities, they are more willing to do so and more confident in their ability.

Which of the following is most likely to help fourth-grade students understand the concept of characterization? A. Having students read and watch a variety of interviews with a noted author B. Having students act out the sounds and movements of various animals from a familiar story C. Directing students in a classroom performance of a play that is based on a familiar story D. Coaching individual students on the use of vocal inflection in reciting stories

Option C is correct because directing students in a classroom performance will help them understand characterization as they act out characters and actions as described by the author. Option A is incorrect because having students read and watch a variety of interviews with a noted author is best suited to helping students learn about that particular author's work. Option B is incorrect because having students act out the sounds and movements of various animals is unlikely to help them understand the concept of characterization without additional instruction. Option D is incorrect because coaching individual students on the use of vocal inflection is more likely to help students understand intonation than characterization.

During a unit on propaganda, a sixth-grade teacher asks students to evaluate short statements made in advertising materials and to attempt to detect any faulty reasoning. Which of the following levels of reading comprehension is primarily being targeted in the lesson? A. Literal B. Inferential C. Evaluative D. Appreciative

Option C is correct because evaluative comprehension includes the skill of detecting faulty reasoning. Students must apply what they have read to their own lives and knowledge of the world to make judgments about text.

Which of the following statements best explains why the algebraic formula for the area of a triangle is 1/2 bh? A. A parallelogram can be transformed into a rectangle if a triangular piece is moved from one side to the other. B. The height of a triangle is not equal to the length of one of its sides, and the length must be divided by 2 to be used to find the area of the triangle. C. A parallelogram is composed of two congruent triangles, so the area of a parallelogram with the same base and height as the triangle can be divided by 2 to find the area of the triangle. D. This formula is only true for scalene triangles because all of their sides are different lengths, so one has to use the base and height to find the area.

Option C is correct because every parallelogram is made up of two congruent triangles. The formula for the area of a parallelogram is b h ¥ ,, so the area of each of the two congruent triangles is half the area of the parallelogram. Two congruent triangles can always be arranged to form a parallelogram with the same base and the same height as the triangles. The area of the triangle will therefore be one-half as much as that of the parallelogram.

Word problem: Samantha's Bakery sells cupcakes in packages of 12 and cookies in packages of 20. The bakery sold the same number of cupcakes and cookies yesterday. What is the minimum number of cupcakes that the bakery could have sold? A teacher creates the word problem shown for a math lesson. Based on the word problem, the lesson will most likely cover which of the following mathematics concepts? A. Least common factor B. Greatest common factor C. Least common multiple D. Greatest common multiple

Option C is correct because finding the least common multiple will identify the smallest number of cupcakes sold. In fact, let x be the number of packages of cupcakes that were sold and y be the number of packages of cookies that were sold. The number of sold cupcakes will be 12x and the number of sold cookies will be 20y. Clearly 12 divides 12x and 20 divides 20y. Since 12 20 , x = y 20 must also divide 12 . x Since 12 and 20 both divide 12x, 12x is a common multiple of 12 and 20. The problem asks for the minimum (or least) number of cupcakes that could have been sold, so the least common multiple must be found to answer the question.

An object is being acted on by a force of 20 N directed to the left and a force of 30 N directed to the right. What is the net force acting on the object? A. 10 N to the left B. 50 N to the left C. 10 N to the right D. 50 N to the right

Option C is correct because forces are vector quantities; thus, both magnitude and direction must be considered when adding. In the example, the forces are acting in opposite directions, so the net magnitude of the force is 10 N. The force directed to the right is greater, so the net force is directed to the right.

To best assess a student's graphophonemic knowledge, a teacher should have the student A. read aloud the letters of the alphabet to a partner. B. circle the vowels in the student's first name. C. identify a word based on a sequence of letter-sound correspondence. D. write all the irregular words the student knows in a spelling journal.

Option C is correct because identifying the letter-sound correspondence will allow students to decode other words.

When teaching students to sing using the Kodály method, which of the following is the first interval taken from folk songs and used in echo singing? A. Ti-do B. Mi-do C. Sol-mi D. La-ti

Option C is correct because it is the first interval that is within the Kodály sequence of intervals

A first-grade teacher plans to use the following passage from a nonfiction text as the introduction for a series of literacy lessons about tigers. As part of the unit, students will use the nonfiction excerpt and other research to create their own stories about a tiger. Passage: The tiger is a large cat found in Asia and parts of Russia. Most tigers are black, white and orange, although some are only white and black. They can run at speeds of 35 miles per hour, even though they are the largest species of cat. Tigers can weigh 400-675 pounds, and their bodies can be up to nine feet long. Is your body as long as a tiger? Tigers are endangered in the wild, because people often destroy their forest homes. In the United States, tigers can be seen in zoos and wildlife preserves. Have you ever seen a tiger? The following is the fictional story written by Jack: Sammy the baby tiger lives at a zoo in Texas. He like to play with his toy ball. He also like to play with his bruther and sistur. Which of the following sentences from the excerpt best explains the idea of cause and effect? A. Most tigers are black, white and orange, although some are only white and black. B. They can run at speeds of 35 miles per hour, even though they are the largest species of cat. C. Tigers are endangered in the wild, because people often destroy their forest homes. D. In the United States, tigers can be seen in zoos and wildlife preserves.

Option C is correct because it provides a cause-and-effect relationship.

Of the following mixtures, which can be most effectively separated into its components using a mesh screen? A. Oil and water B. Sand and water C. Sand and pebbles D. Table sugar and water

Option C is correct because mesh screens or sieves are used to separate solid materials based on particle size. The particle size of sand is less than 2 mm while the particle size of gravel is greater than 2 mm. Therefore a screen with openings of 2 mm can be used to separate sand and gravel.

Students in a kindergarten class are learning to analyze and spell phonetically regular words. They have learned to identify individual phonemes and to blend onsets and rimes. Which of the following is the most appropriate to teach next? A. Vowel-consonant patterns B. Types of syllables C. Short and long vowels D. Vowel diphthongs

Option C is correct because once students have learned the individual sounds made by letters and can blend the sounds at the beginnings and ends of words, they must be able to analyze the vowels within the word and determine whether the vowels are short or long so they can assign meaning to the word as a whole.

Which of the following is the most appropriate tool for teaching students the geographic concept of hemispheres? A. Web-based encyclopedias B. Digital global positioning systems C. Online three-dimensional maps D. Internet travel blogs

Option C is correct because online three-dimensional maps allow students to see Earth from many viewpoints besides traditional flat representations.

A fourth-grade teacher wants to help students become more aware of their literacy development and better monitor their progress in reading and writing. To best help students track their own development, the teacher should have students A. write as many grade-level vocabulary words by memory as they can within five minutes. B. complete a daily journal entry reflecting on what they have learned that day. C. set specific and individual goals regarding improvements they want to make. D. work with a partner to complete weekly running records.

Option C is correct because setting specific and individual goals will help students become more invested in their own learning and will provide them with a way to hold themselves accountable.

While discussing a play they recently saw, students note that one of the characters often appeared onstage in the shadows. When confronted with upsetting news, the same character was shown in red light. As described, which of the following is a primary purpose of the lighting? A. Enhancing performance visibility B. Directing an actor's movement C. Prompting inferences about the character's personality D. Developing a realistic setting for the character

Option C is correct because specifically placed lighting or colored transparencies over the lights helps cue an inference regarding the character's personality.

Which of the following is most likely to occur when a warm air mass meets a cold air mass at Earth's surface? A. The sky will become clear. B. All wind will cease. C. Clouds will form and inclement weather will usually develop. D. The air pressure will increase and fog will form.

Option C is correct because the boundary of air masses of two distinct temperatures is called a front. Along cold fronts and warm fronts, clouds develop as the less dense warm air rises above the cooler air. This allows for the formation of clouds, and usually precipitation occurs.

Third-grade students are assigned to write about their city after reading various texts about it. Which of the following types of writing will best allow the students to include academic vocabulary they learned from the reading? A. Rhyming poetry B. Autobiography C. Expository essay D. Fictional story

Option C is correct because the expository essay is the only writing example given that is not creative in nature. It is nonfiction that easily lends itself to the inclusion of academic vocabulary words.

Which of the following is the stage in the life cycle of a butterfly that is the major period of feeding and growth? A. Adult B. Egg C. Larva D. Pupa

Option C is correct because the larval (caterpillar) stage is the major feeding and growth stage.

A sixth-grade teacher has a number of students experiencing reading difficulties. Which of the following is the best way to assess the students' fluency skills and reading levels? A. Having students read silently and answer questions that follow the reading B. Having students read aloud to determine how fast they can read a piece of text C. Having students read aloud for one minute and count the number of words read correctly D. Having students read silently and write down any unfamiliar words in the text

Option C is correct because the main method of evaluating reading fluency and determining the reading level is to count the number of words a student reads correctly during a one-minute oral reading.

3*(2+6)^2 - 4 [3(2)+3(6)]^2 - 4 (24)^2 - 4 576 - 4 572 A student simplifies the initial expression by applying the rule of order of operations. Which of the following best describes the student's error when simplifying the expression? A. The student added before evaluating the power of the exponents. B. The student evaluated the exponent before subtracting. C. The student multiplied before simplifying within grouping symbols. D. The student simplified the expression from left to right.

Option C is correct because the student multiplied before evaluating within the grouping symbol, violating the rule for order of operations: parentheses left to right, exponents left to right, multiplications or divisions left to right, additions or subtractions left to right.

A second-grade teacher can best help students develop an understanding of a new text by asking students to A. complete a fill-in-the-blank quiz after reading a whole-class story. B. make predictions about a story before they see the cover of the book. C. use literature response journals to record mental images generated from their reading. D. read a similar story aloud to a partner and then answer questions orally.

Option C is correct because when readers create mental images, they engage with text in ways that make it memorable to them.

After reading Charlotte's Web, a student writes the following summary. The most important part of the story is when Charlotte saves the life of Wilbur the pig. She does this by writing words in her web that describe Wilbur. All the towns' people love Wilbur. The person who loves Wilbur the most is probably Fern. She is a human girl who raised Wilbur. The animals in this story talk to one another. In the end, Wilbur takes care of Charlotte's magnum opus. Then, one day they hatch, and he has new friends. 1. Based on the summary, the student will benefit most from instruction in which of the following skills? A. Making personal connections B. Identifying the main character C. Focusing on the key details D. Supporting textual interpretations Which of the following provides the best evidence that Charlotte's Web is a fantasy? A. The main character is a human girl. B. The animals talk to each other. C. The story includes scientific information about spiders. D. The story has a theme about the importance of friendship.

Option C is focusing on the key details. The student is having difficulty narrowing down the summary of the story and is giving too many supporting details. Option B is the animals talk to each other. Animals do not talk in the real world.

"There is an opinion that parties in free countries are useful checks upon the administration of the government and serve to keep alive the spirit of liberty. This within certain limits is probably true; and in governments of a monarchical cast, patriotism may look with indulgence, if not with favor, upon the spirit of party. But in those of the popular character, in governments purely elective, it is a spirit not to be encouraged. From their natural tendency, it is certain there will always be enough of that spirit for every salutary purpose. And there being constant danger of excess, the effort ought to be by force of public opinion, to mitigate and assuage it. A fire not to be quenched, it demands a uniform vigilance to prevent its bursting into a flame, lest, instead of warming, it should consume." In the excerpt above from his final speech as president, George Washington urged United States citizens to A. repay the national debt. B. avoid foreign alliances. C. limit freedom of speech. D. refrain from joining political parties.

Option D is correct because Washington is discussing his beliefs about the dangers of political parties in the United States. Washington believed that the formation of political parties might affect the nation's ability to make effective decisions.

A first-grade teacher plans to use the following passage from a nonfiction text as the introduction for a series of literacy lessons about tigers. As part of the unit, students will use the nonfiction excerpt and other research to create their own stories about a tiger. Passage: The tiger is a large cat found in Asia and parts of Russia. Most tigers are black, white and orange, although some are only white and black. They can run at speeds of 35 miles per hour, even though they are the largest species of cat. Tigers can weigh 400-675 pounds, and their bodies can be up to nine feet long. Is your body as long as a tiger? Tigers are endangered in the wild, because people often destroy their forest homes. In the United States, tigers can be seen in zoos and wildlife preserves. Have you ever seen a tiger? The following is the fictional story written by Jack: Sammy the baby tiger lives at a zoo in Texas. He like to play with his toy ball. He also like to play with his bruther and sistur. In which of the following ways does the expository text best help build students' vocabulary knowledge and development? A. By connecting geographic regions to the topic of the excerpt B. By including advanced words such as "endangered" and "preserve" C. By encouraging students to respond to a question that requires specialized vocabulary D. By allowing students to determine the meaning of unknown words in context

Option D is correct because although the excerpt contains some difficult words ("species" and "endangered"), students should be able to arrive at the meanings of these words by listening to the surrounding words and discussing the context of the sentences and paragraph as a whole

When a vast amount of ash is expelled from a volcano during a catastrophic eruption, some of this ash can remain in the atmosphere for years. Which of the following changes in Earth's system most likely results from this situation? A. Decreased solar reflection leading to higher global temperatures B. Decreased solar reflection leading to lower global temperatures C. Increased solar reflection leading to higher global temperatures D. Increased solar reflection leading to lower global temperatures

Option D is correct because as more ash is added to the atmosphere, sunlight is more liable to bounce off the ash particles and be reflected back into space. When less sunlight passes through the atmosphere, Earth's surface and atmosphere remain cooler

Which of the following terms describes the phenomenon of a young child having greater control of his or her upper torso than of his or her legs or feet? A. Component stages B. Proximodistal development C. Developmental biodynamics D. Cephalocaudal development

Option D is correct because cephalocaudal development describes physical development that starts from the head, then progresses to the lower parts of the body, such as the legs and feet.

Which of the following types of children's songs combine singing and movement in order to promote enjoyment of music and develop music literacy in students? A. Work songs B. Spirituals C. Jingles D. Game songs

Option D is correct because game songs are used throughout the world and by different cultures for play and enjoyment. Game songs often use movement and dance. Learning game songs helps to promote musicality among children and is the basis for becoming musically literate.

A first-grade teacher has set up the following math workstations for students to work in pairs. Station 1: Students toss 7 two-color counters from a cup and record the addition equation represented. They repeat the process ten times. Station 2: Each student builds a tower of 8 cubes using two different colors, and then records the addition equation that the colors represent. Students then exchange towers and record the addition equation for the new towers. Station 3: Students are provided with 9 counters each. One student hides some of the counters. The other student looks at how many counters remain present and determines how many are hidden. The students then record the equation that the missing counters represent. Which of the following relationships are the students most likely exploring in the stations? A. Spatial concepts B. One more and one less C. Benchmarking numbers D. Part-part-whole

Option D is correct because in the activity, the students are conceptualizing that a number is made up of two or more parts.

A first-grade teacher plans to use the following passage from a nonfiction text as the introduction for a series of literacy lessons about tigers. As part of the unit, students will use the nonfiction excerpt and other research to create their own stories about a tiger. Passage: The tiger is a large cat found in Asia and parts of Russia. Most tigers are black, white and orange, although some are only white and black. They can run at speeds of 35 miles per hour, even though they are the largest species of cat. Tigers can weigh 400-675 pounds, and their bodies can be up to nine feet long. Is your body as long as a tiger? Tigers are endangered in the wild, because people often destroy their forest homes. In the United States, tigers can be seen in zoos and wildlife preserves. Have you ever seen a tiger? The following is the fictional story written by Jack: Sammy the baby tiger lives at a zoo in Texas. He like to play with his toy ball. He also like to play with his bruther and sistur. Which of the following comments from the teacher best addresses the strengths and weaknesses of Jack's writing? A. You need to add an illustration to help the reader picture what the tiger cub is doing. B. You did a great job using punctuation at the end of each sentence to organize your thoughts. C. Have you considered using a dictionary to help you identify misspelled words? D. Have you considered adding more details to describe the tiger cub to complement the complete sentences you created?

Option D is correct because it addresses the strengths and needs of Jack's writing.

Which of the following behaviors is most likely to reduce a person's risk of developing melanoma? A. Avoiding cigarettes B. Abstaining from alcohol use C. Avoiding exposure to carcinogenic chemicals D. Protecting the skin against excessive exposure to the sun

Option D is correct because melanoma is a type of skin cancer that is primarily caused by long-term unprotected exposure to the sun.

Which of the following silent reading practices provides the best conditions for students' individual reading improvement? A. Engaging in sustained silent reading for an hour once a week while the teacher reads a book to model engaged reading behaviors B. Dropping everything and reading several times a week for 30 minutes while the teacher grades papers or prepares instructional materials C. Reading silently any time independent work has been completed early while the teacher keeps anecdotal records of student behaviors D. Participating in scaffolded silent reading for 20 minutes every day while the teacher confers with students individually about their reading

Option D is correct because students are receiving support through scaffolding. This could be in the form of minilessons, conferencing, dialogue journaling or other supports.

Which of the following is the Coriolis effect most likely to influence? A. The amount of solar radiation reaching Earth's surface B. The height of a tsunami C. The dew point of an air mass D. The circulation pattern of global wind belts

Option D is correct because the Earth rotates, and as air moves, it undergoes an apparent deflection from its path, as seen by an observer on Earth. This apparent deflection is called the Coriolis effect and is a result of Earth's rotation.

Of the following, which best demonstrates the use of an inquiry-based activity during a unit on physical properties of matter? A. Creating a graphic organizer of physical properties, using a textbook B. Describing the physical properties of different objects in a journal C. Matching terms that describe physical properties with their definitions D. Separating a mixture, using the physical properties of its components

Option D is correct because the activity of separating a mixture into its components engages the students and develops inquiry skills such as conducting investigations, gathering data, analyzing evidence, and logical reasoning.

Celeste is buying erasers for 8 of her friends. There are 76 erasers left at the store. Which of the following approaches can Celeste use to determine the greatest number of erasers she can buy to give each of her friends the same number and have none remaining? A. Drawing a picture of 76 erasers and circling groups of 8 before counting the number of groups created B. Creating a table in which one column represents the number of erasers at the store and the other represents the number of erasers each friend receives C. Using a standard algorithm learned previously in class to solve 76 divided by 8 D. Making an organized list of the multiples of 8 to see which one is closest to 76

Option D is correct because the approach described provides the correct answer of how many erasers Celeste should buy at the store, 72.

A fifth-grade teacher instructs students to use a camera to take pictures of various objects around the school. The students are to use the following methods when taking pictures. • Shoot from up high and down low. • Shoot from different sides and angles. The activity will best help students understand the visual concept of A. balance. B. narrative. C. symbolism. D. perspective.

Option D is correct because the resulting photographs will demonstrate to the students how perspective (the illusion that images diminish in space along converging lines) is affected by different angles.

When teaching students about the lines and spaces of the grand staff, the mnemonic "All Cows Eat Grass" helps them remember A. the lines of the treble clef. B. the spaces of the treble clef. C. the lines of the bass clef. D. the spaces of the bass clef.

Option D is correct because the spaces of the bass clef, beginning with the lowest space, are A-C-E-G. Option A is incorrect because the lines of the treble clef, beginning with the lowest line are E-G-B-D-F. Option B is incorrect because the spaces of the treble clef, beginning with the lowest space, are F-A-C-E. Option C is incorrect because the lines of the bass clef, beginning with the lowest line, are G-B-D-F-A.

Which of the following musical terms tells the performer to play loudly, then immediately play quietly? A. cresc. B. mf C. dim. D. fp

Option D is correct because the symbol means to perform loudly then immediately quietly. Option A is incorrect because the symbol means to play increasingly louder. Option B is incorrect because it means to perform moderately loudly. Option C is incorrect because the symbol means to play increasingly more quietly.

Time 12:05 A.M. 2:05 A.M. 4:05 A.M. 5:05 A.M. 6:05 A.M. Temperature 67 60 53 49.5 46 A cold front arrived at 12:05 A.M., and the subsequent temperatures were recorded in the table shown. If x represents the number of hours after 12:05 A.M., which of the following equations best represents the change in temperature as a function of time? A. y = 3.5x + 67 B. y = -7.1x + 67 C. y = -7.1x - 67 D. y = -3.5x + 67

Option D is correct because the y-intercept represents the starting temperature, which was 67, and the slope represents the rate at which the temperature changed per hour, which was 60-67/2-0 = -3.5

Which of the following is equivalent to 200 centimeters? A. 2 millimeters B. 20 millimeters C. 0.2 meter D. 2 meters

Option D is correct because there are 100 centimeters in 1 meter, therefore 200 centimeters is equivalent to 2 meters.

Which of the following assignments best assesses students' use of quotation marks in writing? A. A journal entry describing the outcome of a science experiment B. A compare and contrast essay about two battles reviewed in social studies class C. A persuasive letter to the school board about the school dress code D. A personal narrative about a time that students disagreed with a friend

Option D is correct because this assignment lends itself to the use of dialogue, and therefore to the use of quotation marks.

Of the following, which is the leading cause of preventable death in the United States? A. Distracted driving B. Infectious diseases C. Sedentary lifestyle D. Tobacco use

Option D is correct because tobacco use, in particular tobacco smoking, is the leading cause of death in the United States and worldwide. Tobacco use is a major risk factor for heart and lung diseases, including lung cancer.

Based on the economic principles of supply and demand, a decrease in the production rates of oil will most likely result in which of the following? A. Increased capital investment in the oil industry B. Increased wages for oil-field workers C. Increased stock of produced oil D. Increased oil prices for consumers

Option D is correct because when supply falls, the temporary shortage drives up the price.

A first-grade teacher has several students who lack phonological and phonemic awareness skills. To best provide opportunities for the students to develop the skills at home, the teacher should assign homework that includes which of the following activities? A. Writing the alphabet on notebook paper B. Listening to stories on tape or on a home computer C. Pointing out the vowels in words when reading aloud D. Playing word and sound games with family members

Option D is correct because word and sound games allow students to hear the individual sounds in words.

An art teacher has her sixth-grade students use a compass, ruler and colored pencils to create a design that will be woven onto a pegged piece of wood in the form of string art as shown below. The activity would be most appropriate for which of the following interdisciplinary units? A. A social studies unit on Medieval stained glass B. A science unit about rain and snow C. A reading unit focused on sequencing ideas D. A math unit on geometric shapes and angles

Option D is correct because wrapping the string on the pegs in different patterns creates corresponding differences in angles and shapes.

Before casting for a class play, a sixth-grade teacher tells the class that the main character will have a monologue during the play. The monologue most directly challenges students to develop which of the following skills? A. Dialoguing with another character B. Using expressive movements C. Improving some of the lines D. Memorizing a long speech

Option D is memorizing a long speech. A monologue is a long speech by a single character that requires memorization and delivery of a long speech.

A teacher reviews four students' current stages of writing development and notes the following. Student 1: The student writes letters and words but sometimes does not add spacing between words. Student 2: The student makes an assortment of marks resembling a drawing on a paper. Student 3: The student spells many words the way they sound. Student 4: The student writes capital letters without spacing. Based on the information above, which TWO students display the most advanced development in their writing? A. Student 1 B. Student 2 C. Student 3 D. Student 4

Options A and C are correct because student 1 displays characteristics of the stage of writing development known as "Beginning Sounds Emerge." Student 3 displays characteristics of the stage of writing development known as "Initial, Middle, and Final Sounds." Both of these stages are more advanced than those being displayed by Student 2 and Student 4.

Which of the following are planets? Select all that apply. A. Mars B. Mercury C. Neptune D. Titan

Options A, B and C are correct because Mars, Mercury and Neptune meet all the conditions set by the International Astronomical Union to be classified as planets. According to the IAU definition, a planet is a celestial body that is in orbit around the Sun, has sufficient mass to assume a nearly round shape, and has cleared the neighborhood around its orbit.

A teacher works with third-grade students to teach them the steps of the writing process. Currently, the students are writing sentences and paragraphs. Which THREE of the following steps of the writing process remain for the students to complete? A. Sharing or displaying the finished product B. Taking out or adding words or sentences C. Rewriting text to correct any mechanical errors D. Brainstorming ideas on a topic

Options A, B and C are correct because these steps of the writing process all take place after a draft of writing is completed.

Which of the following instructional activities will best help students to identify the text features in expository texts? Participating in a scavenger hunt to locate the elements of text Reading and discussing a nonfiction text with a partner Discussing the text and writing a short summary Analyzing the main body of the text

Participating in a scavenger hunt to locate the elements of text

Which of the following is the most appropriate way for a teacher to help students develop oral language skills? Decreasing wait times during verbal conversations Limiting environmental print in the classroom Posting pictures illustrating concepts on a word wall Providing opportunities for interactions between peers

Providing opportunities for interactions between peers

Which of the following practices best supports younger students in the prewriting stage of the writing process? Providing opportunities for students to use oral conversation to organize their thoughts Reviewing prompts and outlines for students to use as they begin their rough drafts Having advanced peers model and explain cognitive processes for brainstorming Planning whole-class instruction on grammar and word usage based on content

Providing opportunities for students to use oral conversation to organize their thoughts

Which of the following teacher actions best supports the development of students' reading vocabulary? Providing students with opportunities to read independently Engaging students in writing about what they read in a journal Having students do multiple readings of the same passage or text Guiding students to select books slightly below their frustration reading level

Providing students with opportunities to read independently

A physical education teacher wants to develop students' appreciation for fair play along with their health-related fitness. Which of the following strategies best supports the teacher's goal during a soccer game? Emphasizing that everyone can be excellent at soccer Diminishing competition and rivalry by not keeping score Recognizing supportive behavior among teammates Singing the school song before the beginning of the game

Recognizing supportive behavior among teammates

Which of the following phonemic-awareness skills will most students master first? Deleting the initial phoneme in "cat" to form "at" Recognizing that the word "man" sounds like "can" and "ran" Substituting the phoneme /n/ for the /t/ in the word "cat" to make "can" Segmenting the word "sat" into three individual phonemes

Recognizing that the word "man" sounds like "can" and "ran"

A second-grade teacher plans a literature circle in which students independently read picture books and then meet in small groups to participate in oral discussions. Which of the following assignments would ensure all key components of a balanced approach to literacy development are addressed? Rereading self-selected sections of the book with a partner Reflecting on text-to-self connections in a writing journal Retelling the story orally using picture sequence cards for support Dramatizing the story using a teacher-created readers' theater script

Reflecting on text-to-self connections in a writing journal

Which of the following activities is the best way for elementary students to learn the techniques of pragmatic language usage? Listening to famous literary monologues Taking notes while watching a documentary Observing local politicians delivering speeches Role-playing a range of sample conversations

Role-playing a range of sample conversations

A teacher is reinforcing a geography lesson by having students watch a video of dancers performing a reel. The music accompanying the dancers is played on the accordion, fiddle, and tin whistle. What country is the teacher focusing on? Israel Scotland Russia Mexico

Scotland

Which of the following most influenced the development of representative government in the United States? The Wealth of Nations, by Adam Smith Leviathan, by Thomas Hobbes Common Sense, by Thomas Paine Second Treatise of Government, by John Locke

Second Treatise of Government, by John Locke

A teacher is planning to demonstrate the concept of multiplication to students for the first time. Which of the following is best for the teacher to do when first introducing the concept? Asking students to draw a picture of the factors in a multiplication problem Writing a numeral to represent each factor in a multiplication problem Giving a multiplication problem to students in abstract form and having them work together to solve it Solving a multiplication problem using some type of concrete manipulative

Solving a multiplication problem using some type of concrete manipulative

hich of the following activities is most effective when assessing students' abilities to use science process skills? Students writing about science process skills in a journal entry Students completing a multiple-choice test about key science process skills Students designing and implementing hands-on tasks to find a solution to a problem Students interpreting a picture that shows a scientist conducting an experiment in a laboratory

Students designing and implementing hands-on tasks to find a solution to a problem

Which of the following statements about media literacy of elementary students is most accurate? Students have an intuitive understanding of media and need minimal instruction in how to develop media-related skills. Students need instruction in using critical-thinking skills to evaluate the function and validity of media. Students quickly realize that most modern media content is created to convey a neutral viewpoint. Students are more proficient at mastering complex content knowledge in a media-rich environment.

Students need instruction in using critical-thinking skills to evaluate the function and validity of media.

Which of the following statements about vocabulary instruction is most closely aligned with research-based best practice? Students should be exposed every day to the greatest possible number of new words. Students should annotate official dictionary definitions of words in personal journals. Students should be exposed to carefully selected words used in multiple contexts. Students should learn words first in isolation before encountering them in authentic contexts.

Students should be exposed to carefully selected words used in multiple contexts.

In which of the following ways does the democratic government of the United States differ from that used by the Greek city-state of Athens? Athenian citizenship was based on land ownership, whereas citizenship in the United States is purely hereditary. The Athenian government was controlled by a single appointed executive official, whereas the United States government is controlled by a single elected executive official. The Athenian assembly allowed all qualified citizens to take part in legislation, whereas representative legislators are elected in the United States. A large percentage of people in Athens were eligible to vote, whereas in the United States very few people are eligible to vote.

The Athenian assembly allowed all qualified citizens to take part in legislation, whereas representative legislators are elected in the United States.

The United States Constitution differed from the Articles of Confederation in which of the following ways? The Constitution decreased the national government's control over the military. The Constitution increased the sovereignty of the states. The Constitution created a separation of powers in the national government. The Constitution gave all states equal representation in the House of Representatives.

The Constitution created a separation of powers in the national government.

Which of the following was primarily responsible for upsetting the balance between free states and slave states prior to the start of the United States Civil War? The Spanish-American War The War of 1812 The Mexican-American War The Texas Revolution

The Mexican-American War

Which of the following activities best supports third-grade students in becoming self-directed critical readers? Producing rhyming words Decoding new words Identifying common punctuation Using graphic organizers

Using graphic organizers

Which of the following strategies will best support a teacher who is instructing students on how to answer literal, inferential, and evaluative comprehension questions? Having students read silently and then answer text-based comprehension questions Asking students to paraphrase the main idea and important details of text Increasing the amount of discussion focused on text-dependent questions Using question starters based on varying levels of Bloom's taxonomy

Using question starters based on varying levels of Bloom's taxonomy

Which of the following types of movement is easiest for kindergarten students when playing music games in the classroom? Walking Hopping Skipping Galloping

Walking

A fourth-grade teacher is planning a lesson in which students will try to identify and list all of the homophones incorporated into a song. The activity best supports students' spelling development by linking spelling to which of the following? Sound blends Word meaning Writing conventions Phonological awareness

Word meaning

Of the following substances, which is the best conductor of electricity? Wood Pure water Oxygen gas Aluminum

aluminum

During group presentations on the structure and function of the federal government, a group of students presents to the class the slide below that contains erroneous information. "The Judicial Branch: This branch uses the court system to enforce laws made by the legislative branch" It would be most appropriate for the teacher to provide the students with additional instruction on the a) functions of the legislative branch b) powers of the executive and judicial branches c) checks of the executive branch d) membership of the judicial and legislative branches

b) powers of the executive and judicial branches

During an activity a first-grade teacher notices that several students cannot decode new words when an onset is added to a rime such as -at. The students would most benefit from further instruction in which of the following literacy skills? Counting syllables Blending phonemes Isolating initial sounds Recognizing vowel digraphs

blending phonemes

A kindergarten class is creating an alphabet tree on a bulletin board in their classroom. As each letter of the alphabet is studied, students bring in examples of things beginning with the letter from their homes. The teacher photographs the items and adds the images to the appropriately labeled branch of the alphabet tree. This activity best promotes the development of alphabetic knowledge through cooperative learning groups. teacher modeling. collaboration with families. anchor activities.

collaboration with families.

position of Earth during a full moon

directly between the sun and the moon

Planning and implementing physical education activities for students with disabilities in the least restricted environment can most accurately be accomplished by gearing activities toward each student's unique abilities to the greatest extent possible. offering an alternative program for students with disabilities. recommending students with disabilities participate as activity scorekeeper. providing private instruction for students with disabilities.

gearing activities toward each student's unique abilities to the greatest extent possible.

By engaging students in the follow-up activity, the teacher best demonstrates an understanding of the importance of using developmentally appropriate exercises that enhance students' manipulative skills. nonlocomotor skills. rhythmic skills. locomotor skills.

manipulative skills.

Which of the following types of rocks are formed under conditions of intense heat and pressure but not enough heat to melt the rock material? Shale Igneous Sedimentary Metamorphic

metamorphic

The last skill a teacher should expect students to master during a unit on phonemic awareness is phoneme deletion. syllable deletion. blending phonemes. blending onsets and rimes.

phoneme deletion.

The following is a writing excerpt from a first grader. The Katipilr was haging on the tree banch. It gru and gru then it bkam a Butifl Buttrfli. Which of the following stages of the writing workshop process is the student ready to initiate next? A) publishing B) prewriting C) revising D) drafting

revising

A student whose oral reading is labored and choppy struggles to comprehend grade-level texts independently. To improve the student's reading comprehension, the teacher should provide additional instruction primarily focused on phonological and phonemic awareness. sight-word automaticity and fluency. vocabulary development and oral language. alphabetic awareness and phonics.

sight-word automaticity and fluency.

A second-grade teacher notices that several students are experiencing difficulty decoding words like "fleet," "mean," and "coat." In which of the following areas should the teacher plan further instruction for the students? Open syllables Vowel digraphs Consonant blends Inflectional endings

vowel digraphs


Kaugnay na mga set ng pag-aaral

ну филка же, как мы любим

View Set

ATI Engage Fundamentals (Psychosocial Concepts for Nursing Practice) Grief

View Set

Peds Chapter 24: Genitourinary Disorders - ML5

View Set

Business Policy and Strategy - Chapter 1 - Basic Concepts of Strategic Management

View Set

Unit 10: Taxation of Life Insurance & Annuities Checkpoint Exam

View Set